Question and Answers Forum

All Questions      Topic List

Others Questions

Previous in All Question      Next in All Question      

Previous in Others      Next in Others      

Question Number 52214 by Tawa1 last updated on 04/Jan/19

Commented by Tawa1 last updated on 04/Jan/19

Evaluate

$$\mathrm{Evaluate} \\ $$

Commented by mr W last updated on 05/Jan/19

i know it′s (9/(32))  but i can′t prove.

$${i}\:{know}\:{it}'{s}\:\frac{\mathrm{9}}{\mathrm{32}} \\ $$$${but}\:{i}\:{can}'{t}\:{prove}. \\ $$

Commented by rahul 19 last updated on 05/Jan/19

Yes Sir! U r right . Just interchange the summation (as they are independent) , call them as EQ. (1) & (2) and add them...��

Commented by Tawa1 last updated on 05/Jan/19

Please show it sir

$$\mathrm{Please}\:\mathrm{show}\:\mathrm{it}\:\mathrm{sir} \\ $$

Answered by tanmay.chaudhury50@gmail.com last updated on 05/Jan/19

T_(m,n) =((m/3^m )/(((3^m /m)+(3^n /n))))  since both m→∞  and n→∞  and  value of (3^m /m)=(3^n /n) when we put m=1,2,3...  n=1,2,3...simutaneously  so   T_(m,n) =T_(r ) =((r/3^r )/(2×(3^r /r)))=(1/2)×((r/3^r ))^2   s=T_1 +T_2 +T_3 +...∞  S=(1/2)[((1/3^1 ))^2 +((2/3^2 ))^2 +((3/3^3 ))^2 +...∞  wait...

$${T}_{{m},{n}} =\frac{\frac{{m}}{\mathrm{3}^{{m}} }}{\left(\frac{\mathrm{3}^{{m}} }{{m}}+\frac{\mathrm{3}^{{n}} }{{n}}\right)} \\ $$$${since}\:{both}\:{m}\rightarrow\infty \\ $$$${and}\:{n}\rightarrow\infty \\ $$$${and}\:\:{value}\:{of}\:\frac{\mathrm{3}^{{m}} }{{m}}=\frac{\mathrm{3}^{{n}} }{{n}}\:{when}\:{we}\:{put}\:{m}=\mathrm{1},\mathrm{2},\mathrm{3}... \\ $$$${n}=\mathrm{1},\mathrm{2},\mathrm{3}...{simutaneously} \\ $$$${so}\: \\ $$$${T}_{{m},{n}} ={T}_{{r}\:} =\frac{\frac{{r}}{\mathrm{3}^{{r}} }}{\mathrm{2}×\frac{\mathrm{3}^{{r}} }{{r}}}=\frac{\mathrm{1}}{\mathrm{2}}×\left(\frac{{r}}{\mathrm{3}^{{r}} }\right)^{\mathrm{2}} \\ $$$$\boldsymbol{{s}}=\boldsymbol{{T}}_{\mathrm{1}} +{T}_{\mathrm{2}} +{T}_{\mathrm{3}} +...\infty \\ $$$${S}=\frac{\mathrm{1}}{\mathrm{2}}\left[\left(\frac{\mathrm{1}}{\mathrm{3}^{\mathrm{1}} }\right)^{\mathrm{2}} +\left(\frac{\mathrm{2}}{\mathrm{3}^{\mathrm{2}} }\right)^{\mathrm{2}} +\left(\frac{\mathrm{3}}{\mathrm{3}^{\mathrm{3}} }\right)^{\mathrm{2}} +...\infty\right. \\ $$$${wait}... \\ $$

Commented by mr W last updated on 05/Jan/19

it′s not correct sir.  you can not transfer T_(∞,∞)  to T_(m,n) .  Σ_(m=1) ^∞  Σ_(n=1) ^∞ a_(m,n) =(a_(1,1) +a_(1,2) +...)+(a_(2,1) +a_(2,3) +...)+(.....)+  =(∞ numbers)+(∞ numbers)+(...)+  =∞×∞ numbers    e.g.  Σ_(m=1) ^∞  Σ_(n=1) ^∞ (1/(m^2 +n^2 ))≠(1/2)Σ_(r=1) ^∞ (1/r^2 )    to check:  S=(1/2)[((1/3^1 ))^2 +((2/3^2 ))^2 +((3/3^3 ))^2 +...∞=((45)/(512))≠(9/(32))

$${it}'{s}\:{not}\:{correct}\:{sir}. \\ $$$${you}\:{can}\:{not}\:{transfer}\:{T}_{\infty,\infty} \:{to}\:{T}_{{m},{n}} . \\ $$$$\underset{{m}=\mathrm{1}} {\overset{\infty} {\sum}}\:\underset{{n}=\mathrm{1}} {\overset{\infty} {\sum}}{a}_{{m},{n}} =\left({a}_{\mathrm{1},\mathrm{1}} +{a}_{\mathrm{1},\mathrm{2}} +...\right)+\left({a}_{\mathrm{2},\mathrm{1}} +{a}_{\mathrm{2},\mathrm{3}} +...\right)+\left(.....\right)+ \\ $$$$=\left(\infty\:{numbers}\right)+\left(\infty\:{numbers}\right)+\left(...\right)+ \\ $$$$=\infty×\infty\:{numbers} \\ $$$$ \\ $$$${e}.{g}. \\ $$$$\underset{{m}=\mathrm{1}} {\overset{\infty} {\sum}}\:\underset{{n}=\mathrm{1}} {\overset{\infty} {\sum}}\frac{\mathrm{1}}{{m}^{\mathrm{2}} +{n}^{\mathrm{2}} }\neq\frac{\mathrm{1}}{\mathrm{2}}\underset{{r}=\mathrm{1}} {\overset{\infty} {\sum}}\frac{\mathrm{1}}{{r}^{\mathrm{2}} } \\ $$$$ \\ $$$${to}\:{check}: \\ $$$${S}=\frac{\mathrm{1}}{\mathrm{2}}\left[\left(\frac{\mathrm{1}}{\mathrm{3}^{\mathrm{1}} }\right)^{\mathrm{2}} +\left(\frac{\mathrm{2}}{\mathrm{3}^{\mathrm{2}} }\right)^{\mathrm{2}} +\left(\frac{\mathrm{3}}{\mathrm{3}^{\mathrm{3}} }\right)^{\mathrm{2}} +...\infty=\frac{\mathrm{45}}{\mathrm{512}}\neq\frac{\mathrm{9}}{\mathrm{32}}\right. \\ $$

Commented by tanmay.chaudhury50@gmail.com last updated on 05/Jan/19

thank you sir...i viewed it diffdrently...

$${thank}\:{you}\:{sir}...{i}\:{viewed}\:{it}\:{diffdrently}... \\ $$

Answered by rahul 19 last updated on 05/Jan/19

S = Σ_(m=1) ^∞  Σ_(n=1) ^∞  ((m^2 n)/(3^m (n.3^m +m.3^n )))  S =  Σ_(n=1) ^∞ Σ_(m=1) ^∞ ((n^2 m)/(3^n (m.3^n +n.3^m )))  ⇒ 2S = ΣΣ((mn)/(n.3^m +m.3^n ))[(m/3^m )+(n/3^n )].  ⇒2S= Σ_(m=1) ^∞ (m/3^m )  . Σ_(n=1) ^∞ (n/3^n )  ⇒2S= (3/4) × (3/4)  ⇒ S = (9/(32 )) .

$${S}\:=\:\underset{{m}=\mathrm{1}} {\overset{\infty} {\sum}}\:\underset{{n}=\mathrm{1}} {\overset{\infty} {\sum}}\:\frac{{m}^{\mathrm{2}} {n}}{\mathrm{3}^{{m}} \left({n}.\mathrm{3}^{{m}} +{m}.\mathrm{3}^{{n}} \right)} \\ $$$${S}\:=\:\:\underset{{n}=\mathrm{1}} {\overset{\infty} {\sum}}\underset{{m}=\mathrm{1}} {\overset{\infty} {\sum}}\frac{{n}^{\mathrm{2}} {m}}{\mathrm{3}^{{n}} \left({m}.\mathrm{3}^{{n}} +{n}.\mathrm{3}^{{m}} \right)} \\ $$$$\Rightarrow\:\mathrm{2}{S}\:=\:\Sigma\Sigma\frac{{mn}}{{n}.\mathrm{3}^{{m}} +{m}.\mathrm{3}^{{n}} }\left[\frac{{m}}{\mathrm{3}^{{m}} }+\frac{{n}}{\mathrm{3}^{{n}} }\right]. \\ $$$$\Rightarrow\mathrm{2}{S}=\:\underset{{m}=\mathrm{1}} {\overset{\infty} {\sum}}\frac{{m}}{\mathrm{3}^{{m}} }\:\:.\:\underset{{n}=\mathrm{1}} {\overset{\infty} {\sum}}\frac{{n}}{\mathrm{3}^{{n}} } \\ $$$$\Rightarrow\mathrm{2}{S}=\:\frac{\mathrm{3}}{\mathrm{4}}\:×\:\frac{\mathrm{3}}{\mathrm{4}} \\ $$$$\Rightarrow\:{S}\:=\:\frac{\mathrm{9}}{\mathrm{32}\:}\:. \\ $$

Commented by mr W last updated on 05/Jan/19

very tricky sir!

$${very}\:{tricky}\:{sir}! \\ $$

Terms of Service

Privacy Policy

Contact: info@tinkutara.com